1. Write the polynomial function that models the given situation.A rectangle has a length of 12 units and a width of 11 units. Squares of x by x units are cut out of each corner, and then the sides are folded up to create an open box. Express the volume V of the box as a polynomial function in terms of x.
2. Write the polynomial function that models the given situation. A square has sides of 24 units. Squares x + 1 by x + 1 units are cut out of each corner, and then the sides are folded up to create an open box. Express the volume V of the box as a function in terms of x.
3. Write the polynomial function that models the given situation. A cylinder has a radius of x + 6 units and a height 3 units more than the radius. Express the volume V of the cylinder as a polynomial function in terms of x.

Answers

Answer 1

Answer:

1.  (12 - 2x)(11 - 2x)x

2. 4(11 - 2x)²(x + 1)

3. π(x³ + 15x² + 63x + 81)

Step-by-step explanation:

1. Write the polynomial function that models the given situation.

A rectangle has a length of 12 units and a width of 11 units. Squares of x by x units are cut out of each corner, and then the sides are folded up to create an open box. Express the volume V of the box as a polynomial function in terms of x.

Since the length of the rectangle is 12 units and its width 11 units and squares of x by x units are cut from its corners, it implies that a length x is cut from each side. So, the length of the open box is L = 12 - 2x and its width is w = 11 - 2x.

Since the cut corners of the rectangle are folded, the side x which is cut represents the height of the open box, h. so, h = x

So, the volume of the open box V = LWh = (12 - 2x)(11 - 2x)x

2. Write the polynomial function that models the given situation. A square has sides of 24 units. Squares x + 1 by x + 1 units are cut out of each corner, and then the sides are folded up to create an open box. Express the volume V of the box as a function in terms of x.

Since the square has sides of 24 units and squares of x + 1 by x + 1 units are cut from its corners, it implies that a length x + 1 is cut from each corner and the length 2(x + 1) is cut from each side. So, the length of side open box is L = 24 - 2(x + 1) = 24 - 2x - 2 = 24 - 2 - 2x = 22 - 2x = 2(11  - x)

Since the cut corners of the square are folded, the side x + 1 which is cut represents the height of the open box, h. so, h = x + 1

Since the area of the base of the pen box is a square, its area is L² = [2(11 - 2x)]²

So, the volume of the open box V = L²h = [2(11 - 2x)]²(x + 1) = 4(11 - 2x)²(x + 1)

3. Write the polynomial function that models the given situation. A cylinder has a radius of x + 6 units and a height 3 units more than the radius. Express the volume V of the cylinder as a polynomial function in terms of x.

The volume of a cylinder is V = πr²h where r = radius and h = height of cylinder.

Given that r = x + 6 and h is 3 units more than r, h = r + 3 = x + 6 + 3 = x + 9

So, V = πr²h

V = π(x + 3)²(x + 9)

V = π(x² + 6x + 9)(x + 9)

V = π(x³ + 6x² + 9x + 9x² + 54x + 81)

V = π(x³ + 15x² + 63x + 81)


Related Questions

The population model given dP/dt â P or dP dt = kP (1)

fails to take death into consideration; the growth rate equals the birth rate. In another model of a changing population of a community it is assumed that the rate at which the population changes is a net rate that is, the difference between the rate of births and the rate of deaths in the community. Determine a model for the population P(t) if both the birth rate and the death rate are proportional to the population present at time t > 0.

Answers

Answer:

.

Step-by-step explanation:

DVD Video Rentals (Refer to Example 3.) The func-
tion V computes the percent share of disc DVD rentals
accounted for by various companies. This function is
defined by V(R) = 37, V(N) = 30, and V(S) = 17,
where R is Redbox, N is Netflix, and S is rental stores.
(Source: Business Insider.)
(a) Write V as a set of ordered pairs.
(b) Give the domain and range of V.
T

Answers

Answer:

[tex](a)\ V = \{(N,30),(R,37),(S,17)\}[/tex]

[tex](b)[/tex]

[tex]Domain = \{N,R,S\}[/tex]

[tex]Range = \{37,30,17\}[/tex]

Step-by-step explanation:

Given

[tex]V(R) = 37,\ V(N) = 30,\ V(S) = 17[/tex]

Solving (a): Set of ordered pair

A function y = f(x) is represented as (x,y)

So, the ordered pair of V is:

[tex]V = \{(R,37),(N,30),(S,17)\}[/tex]

Order the alphabets in increasing order

[tex]V = \{(N,30),(R,37),(S,17)\}[/tex]

Solving (b): The domain and the range

In a function [tex]\{(x_1,y_1),...,(x_n,y_n)\}[/tex]

The domain and the range are represented as:

[tex]Domain = \{x_1,x_2....x_n\}[/tex]

[tex]Range = \{y_1,y_2....y_n\}[/tex]

So, we have:

[tex]Domain = \{N,R,S\}[/tex]

[tex]Range = \{37,30,17\}[/tex]

What fraction is equivalent to eight tentHs

Answers

8/10 is equivalent to 4/5. just divide both 8 and 10 by 2

An ice cream truck started the day with 454545 ice cream sandwiches. During a stop at a busy park they sold sss ice cream sandwiches. After the stop, the truck had 212121 ice cream sandwiches remaining.
Write an equation to describe this situation.
How many ice cream sandwiches did the truck sell at the park?

Answers

Answer:

45- s= 21

s= 45- 21

=24

so the answer is 24

help, please!
find x

Answers

This shape consists of two primary special triangles, 30-60-90 and 45-45-90.

the side with the 60° will be half of the hypotenuse because 30° is across from it.

the side with the 30° will be 5.5√3 because 60° is across from it, and any sides that has 60° across from it is associated with a √3.

x is 5.5√6 because any sides that is "below" the 45° angles are associated with a √2 times the side that's above the 45°

Julie wanted to find out the approximate percentage of non-traditional female college students at public universities. She decided to randomly select 25 females from each university in her state and collect and analyze the data.

Select the statement that is TRUE.


The 25 females that Julie interviews are the sample, and the females that Julie does not interview are the population.


The 25 females that Julie interviews are the sample, and all female college students at state universities are the population.


All female college students at state universities are the sample, and the 25 females that Julie interviews are the population.


The female college students at state universities are the sample, and the female college students at all universities in the United States are the population.

Answers

To solve this question, we need to understand sample and population concepts.

From this, the answer is:

The 25 females that Julie interviews are the sample, and all female college students at state universities are the population.

Example:

I want to estimate the proportion of New York state residents who are Buffalo Bills fans. So i ask, lets say, 1000 randomly selected New York state residents whether they are Buffalo Bills fans, and expand this to the entire population of New York State residents, which is the population of the study.

The sample will be: 1000 randomly selected New York state residents

The population will be: All New York state residents

In this question:

Sample: 25 females from each university in her state.

Population: All female college students in her student.

Thus, the correct statement is:

The 25 females that Julie interviews are the sample, and all female college students at state universities are the population.

For a similar example, you can check https://brainly.com/question/24331410

Martha, Lee, Nancy, Paul, and Armando have all been invited to a dinner party. They arrive randomly, and each person arrives at a different time.

a. In how many ways can they arrive?

b. In how many ways can Martha arrive first and Armando last?

c. Find the probability that Martha will arrive first and Armando last.

Show your work

Answers

Answer:

a) 120

b) 6

c) 1/20

Step-by-step explanation:

a) 5! = 120

b) (5 - 2)! = 6

c) 6/120 = 1/20

find the range of values of a for which 11- 2a>1 is ____

Answers

Answer:

a<5

Step-by-step explanation:

11-2a>1

-2a>1-11

-2a>-10

a<5

The values of y when x2 - 3 y2 = 1 are not only found among its terms, but the numbers x and y are.
74 pages·4 MB

the product of two rational number is _______ number
give answers this question​

Answers

Answer:

rational number

Step-by-step explanation:

multiplying two rationals is the same as

multiplying two such fractions, which will

result in another fraction of this same form

since integers are closed under

multiplication. Thus, multiplying two rational numbers produces another rational number.

"The product of two rational number is rational" So, multiplying two rationals is the same as multiplying two such fractions, which will result in another fraction of this same form since integers are closed under multiplication.Sep 5, 2020

The dotplot below displays the difference in scores for 18 games between a high school soccer team and its opponent.

A dotplot titled difference in Score. A number line going from negative 2 to 4 is labeled Team score minus opponent score. Negative 2, 1; negative 1, 2; 0, 2; 1, 3; 2, 6; 3, 3; 4, 1.

Which of the following is the best explanation for the dots at –1?

In one game, the team beat the opponent by 1 goal.
In two games, the team beat the opponent by 1 goal.
In one game, the team lost to the opponent by 1 goal.
In two games, the team lost to the opponent by 1 goal.

Answers

Answer: Choice D

In two games, the team lost to the opponent by 1 goal.

========================================================

Explanation:

Negative scores indicate the team lost, and the absolute value of those values represent how much of a loss.

So a difference of -1 means the team lost by 1 point.

We have 2 dots over this value, so there are 2 occasions where the team lost  to the opponent by 1 goal.

An example would be that say the team scored 3 goals and the opponent scored 4 goals. So we have a differential of 3-4 = -1. The order is important because we would not say 4-3 = 1.

Answer:

d

Step-by-step explanation:

took the test

PLEASE HELP 40 POINTS
Express each logarithm in terms of ln 3 and ln 5.
ln 81 / 125

Answers

I’ll recommend b or c so choose answer c

The solution is 4ln3 - 3ln5, which is correct option(C).

What are the properties of logarithms?

There are four basic properties of logarithms:

logₐ(xy) = logₐx + logₐy.

logₐ(x/y) = logₐx - logₐy.

logₐ(xⁿ) = n logₐx.

logₐx = logₓa / logₓb.

What are Arithmetic operations?

Arithmetic operations can also be specified by the subtract, divide, and multiply built-in functions.

The operator that perform arithmatic operation are called arithmatic operators .

Operators which let do basic mathematical calculation

+ Addition operation : Adds values on either side of the operator.

For example 4 + 2 = 6

- Subtraction operation : Subtracts right hand operand from left hand operand.

for example 4 -2 = 2

* Multiplication operation : Multiplies values on either side of the operator

For example 4*2 = 8

/ Division operation : Divides left hand operand by right hand operand

For example 4/2 = 2

According to the problem, we will use some of the basic logarithmic properties,

Given expression,

⇒ ln81/125

⇒ ln81 - ln125   [used property : logₐ(x/y) = logₐx - logₐy]

⇒ ln3⁴ - ln5³    [ 81 =3⁴ and 125 = 5³ ]

⇒ 4ln3 - 3ln5   [used property : logₐ(xⁿ) = nlogₐx]

Hence, the solution is 4ln3 - 3ln5

Learn more logarithmic properties here:

brainly.com/question/24211708

#SPJ2

(5.5 X10^6 + 6.3 X10^6)2

Answers

Answer:

(5.5×10⁶+6.3×10⁶)×2

= (5.5+6.3)×10⁶×2

= 11.8×10⁶×2

= 23,600,000

What is the solution to 4x + 2 = 6(-2x - 5) ?
O2
O 16
0-2
O -16
Please need help on this

Answers

x= -2
............
...........
&/&/&:&/&/&

4x+2=6(-2x-5)

<=>4x+2=-12x-30

<=>16x+32=0

<=>x=-2

USE THE PRESENT VALUE FORMULA TO CALCULATE THE AMOUNT OF MONEY THAT MUST BE INVESTED NOW AT 9% ANNUALLY COMPOUNDED QUARTERLY TO OBTAIN 1,000 IN 4 YEARS.

Answers

Answer:

The amount of money that must be invested is $252.

Step-by-step explanation:

Present value formula:

The present value formula is given by:

[tex]P = \frac{F}{(1+r)^n}[/tex]

In which:

P is the present value.

F is the future value.

r is the interest rate.

n is the number of periods.

9% ANNUALLY

This means that [tex]r = 0.09[/tex]

COMPOUNDED QUARTERLY TO OBTAIN 1,000 IN 4 YEARS.

Obtain 1000 means that [tex]F = 1000[/tex]

Compounded quarterly in 4 years, so 4*4 = 16 periods and [tex]n = 16[/tex].

Amount of money that must be invested:

[tex]P = \frac{F}{(1+r)^n}[/tex]

[tex]P = \frac{1000}{(1+0.09)^{16}}[/tex]

[tex]P = 252[/tex]

The amount of money that must be invested is $252.

Which function below has the following domain and range?
Domain: { – 7, - 5, 2, 6, 7}
Range: {0, 1, 8)
o {(1, – 5), (8, 7), (0, 6), (1, – 7), (0, 2)}
o {(-7,0), ( – 5,1), (2, 8), (6,5), (7, 7)}
o {(-7,7), (0,8)}
o {(2,0), ( – 5, 1), (7,8), (6,0), ( – 7, 1)}

Answers

The function that has the given domain and range is: D. {(2,0), ( – 5, 1), (7,8), (6,0), ( – 7, 1)}.

How to Determine the Domain and Range of a Function?

A function is a relation that consists of a domain (a set of x-values) and a corresponding range (set of y-values).

Thus, each ordered pair in a function is expressed (x, y), where all values of x make up the domain, while tehri corresponding y-values make up the range of the function.

Thus, for a function given as,  {(2,0), ( – 5, 1), (7,8), (6,0), ( – 7, 1)}:

The set of x-values is, -7, -5, 2, 6, and 7. Therefore, the domain is:  { – 7, - 5, 2, 6, 7}.

The set of y-values is: 0, 1, and 8. Therefore, the range is:  {0, 1, 8).

Thus, the function that has the given domain and range is: D. {(2,0), ( – 5, 1), (7,8), (6,0), ( – 7, 1)}.

Learn more about domain and range of a function on:

https://brainly.com/question/10197594

#SPJ1

Answer: (2,0), ( – 5, 1), (7,8), (6,0), ( – 7, 1)

Step-by-step explanation: TCB

Scientists have increased their ability to make observations beyond their own senses through the invention of specialized equipment such as Xray telescopes.

Which best explains how such technology has affected society?


Technology has replaced humans' need for independent observation and thought.
Technology has replaced humans' need for independent observation and thought.

Technology has replaced the need for the experimentation in the scientific process.
Technology has replaced the need for the experimentation in the scientific process.

Human knowledge of the universe is limited by ineffective technology.
Human knowledge of the universe is limited by ineffective technology.

Human knowledge of the universe beyond Earth has increased.

Answers

Answer:

D

Step-by-step explanation:

A is rather a strange choice. The tools are used as extensions of our  senses. That's all that technology has done. We still need to observe things for ourselves if we are to learn. I actually really do not know what this statement means. My answer reflects what I think is true. Our need to observe and think is not hindered.

B is never going to be true. We still have to test things out, even with the best tools that we have.

C: The exact opposite is true. Our knowledge of the universe has expanded beyond our belief with the better tools of technology that we have.

D: This is the opposite of C and is the answer.

(SAT PREP) Find the value of x in each of the following excersises

Answers

Answer:

The answer is 155.

Step-by-step explanation:

We can find the remaining parts of the triangle angles.

a man spends RS 608 a month. If he earns Rs 640, what percentage of his invome does he save??.


Please explanation​

Answers

Answer:

5%

Step-by-step explanation:

given,

Earns= Rs 640

spends= Rs 608

saves= (Rs 640 - Rs 608)

=Rs 32

therefore, 32/640x100

answer = 5%

Answer From Gauth Math

Answer:

5%

Step-by-step explanation:

save=640-608=32

(32/640)*100%

5%

1 poir
Question 1. Jessica has $1,625.00 to purchase a five-year Certificate of
Deposit (CD). In the chart, there are CD rates frombankrate.com. What
would the account ending balance be at Synchrony Bank if it is
compounded quarterly? *
Use the Compound Interest Formula to calculate the ending balance. A = P(1 + 5)nt
Nationwide
Bank
Nationwide
2.01%
No
Synchrony Bank
all synchrony
1.95%

Answers

9514 1404 393

Answer:

  $1790.99

Step-by-step explanation:

Given:

  $1625 is invested at an annual rate of 1.95% compounded quarterly for 5 years

Find:

  the ending balance

Solution:

The compound interest formula applies.

  FV = P(1 +r/n)^(nt) . . . Principal P at rate r for t years, compounded n per year

  FV = $1625(1 +0.0195/4)^(4·5) = $1625(1.004875^20) ≈ $1790.99

The account ending balance would be $1790.99.

what Is the si unit of temperature​

Answers

Answer:

the Si unit of temprature in Kelvin (K)

Step-by-step explanation:

Answer:

The answer is Kelvin (k).

Step-by-step explanation:

The kelvin (K) is defined by taking the fixed numerical value of the Boltzmann constant k to be [tex]1.380649*10^{-23}[/tex] when expressed in the unit of joule per kelvin. The temperature 0 K is commonly referred to as "absolute zero." On the widely used Celsius temperature scale, water freezes at 0 °C and boils at about 100 °C. One Celsius degree is an interval of 1 K, and zero degrees Celsius is 273.15 K. An interval of one Celsius degree corresponds to an interval of 1.8 Fahrenheit degrees on the Fahrenheit temperature scale.

The kelvin is also the fundamental unit of the Kelvin scale, an absolute temperature scale named for the British physicist William Thomson (known as Lord Kelvin). An absolute temperature scale has as its zero point absolute zero (−273.15° on the Celsius temperature scale and −459.67° on the Fahrenheit temperature scale), the theoretical temperature at which the molecules of a substance have the lowest energy; hence, all values on such a scale are nonnegative.  

Find the difference between the result of one sixth of product of 10 and 3 multiplied by 4 and 30
Pleas helppppp ;_-((((((

Answers

Answer:

10 or -10,

Read the explanation

Step-by-step explanation:

Rewrite this problem as a numerical expression. As per the wording of this problem, there can be two expressions derived.

1.     [tex]((\frac{1}{6}(10*3)*4)-30[/tex]

2.     [tex]30-((\frac{1}{6}(10*3)*4)[/tex]

Simplify, remember the order of operations. The order of operations is the sequence by which one is supposed to perform operations in a numerical expression. This order is the following:

1. Parenthesis

2. Exponents

3. Multiplication or division

4. Addition or Subtraction

Use this sequence when simplifying and solving the expression:

Expression 1

[tex]((\frac{1}{6}(10*3)*4)-30\\\\=((\frac{1}{6}(30)*4)-30\\\\=(5*4)-30\\\\=20 - 30\\\\= -10[/tex]

Expression 2

[tex]30-((\frac{1}{6}(10*3)*4)\\\\=30-((\frac{1}{6}(30)*4)\\\\=30-(5*4)\\\\= 30-20\\\\= 10[/tex]

Select the correct expressions and value.
Identify the expressions and the value that are equivalent to 6 times 5 squared.
5x 6²
6 x 5
192
6 x 5 x 5
6x5x2
150
6 + 5 x 5
Reset
Next

Answers

Answer:

✔️6 × 5²

✔️6 × 5 × 5

✔️150

Step-by-step explanation:

6 times 5 squared is written as 6 × 5²

Thus:

6 × 25 = 150

Evaluate each of the expressions given to determine whether they are equivalent to 150 or not

✔️5 × 6² = 5 × 36 = 180 (NOT EQUIVALENT)

✔️6 × 5² = 6 × 25 = 150 (EQUIVALENT)

✔️192 ≠ 150 (NOT EQUIVALENT)

✔️6 × 5 × 5 = 6 × 25 = 150 (EQUIVALENT)

✔️6 × 5 × 2 = 6 × 10 = 60 (NOT EQUIVALENT)

✔️150 = 150 (EQUIVALENT)

✔️6 + 5 × 5 = 6 + 25 = 31 (NOT EQUIVALENT)

If side A is 10 inches long, and side B is 24 inches, find the length of the unknown side.

Answers

Step-by-step explanation:

Right Triangles and the Pythagorean Theorem. The Pythagorean Theorem, a2+b2=c2, a 2 + b 2 = c 2 , can be used to find the length of any side of a right triangle.

14. In a garden 746496 apple trees are arranged in such a way that, there are as inany rows as there are in a row. How many rows are there in the garden ​

Answers

Answer:

864

Step-by-step explanation:

do the square root of the total number

e lifetimes of lightbulbs of a particular type are normally distributed with a mean of290 hours and astandard deviation of6 hours. What percentage of the bulbs have lifetimes that lie within 1 standarddeviation to either side of the mean

Answers

Answer:

Step-by-step explanation:

[tex]p(\overline{X}-\sigma \leq X \leq \overline{X}+\sigma)\\\\=p(\dfrac{\overline{X}-\sigma -\overline{X} }{\sigma} \leq Z \leq \dfrac{\overline{X}+\sigma -\overline{X} }{\sigma} )\\\\=p ( -1 \leq Z \leq 1)\\\\=2*(\ p (Z \leq 1)-0.5)\\\\=2*(0.8413-0.5)\\\\=0.6826\\\\\approx{68\%}[/tex]

i need help with this question asapppppp

Answers

9514 1404 393

Answer:

  $11,680.58

Step-by-step explanation:

Usually, I would say copy the example, using 70,000 instead of 55,000. However, the example you show has a couple of errors in it. You need to do what it says, not follow what it did.

__

The first 48,535 is taxed at 15%, so the tax is 0.15×48535 = 7280.25.

The next (70,000 -48,535) = 21,465 is taxed at 20.5%, so the tax is ...

  0.205×21,465 = 4400.325 ≈ 4400.33

The the total tax due on $70,000 is ...

  $7280.25 +4400.33 = $11,680.58 . . . . tax due on $70,000

_____

Additional comments

The example shown has a couple of errors. The tax on the excess amount is figured at 2.05%, not 20.5%, and the 132.53 value from that is shown as 132.23.

__

Any tax table like this one can be reduced to a set of simpler formulas. Here are the formulas for the brackets shown in your tax table.

  ≤ 48535 -- income × 0.15

  ≤ 97069 -- income × 0.205 -2669.425

  ≤ 150,473 -- income × 0.26 -8008.22

  ≤ 214,368 -- income × 0.29 -12,522.41

  > 214,368 -- income × 0.33 -21,097.13

In this case, the second row of this simpler table would give the tax on $70,000 as ...

  tax = 70,000 × 0.205 -2669.425

  tax = 14350 -2669.425 = 11680.575 ≈ 11,680.58 . . . same as above

Each time Kristine gets paid, she spends $20 and saves the rest. If the amount Kristine earns is represented by x and the amount she saves is represented by y, which graph models her savings?
A graph titled Kristine's Savings has money earned on the x-axis and money saved on the y-axis. A line goes through points (5, 100) and (10, 200).
A graph titled Kristine's Savings has money earned on the x-axis and money saved on the y-axis. A line goes through points (20, 1) and (40, 2).
A graph titled Kristine's Savings has money earned on the x-axis and money saved on the y-axis. A line goes through points (0, 20) and (10, 30).
A graph titled Kristine's Savings has money earned on the x-axis and money saved on the y-axis. A line goes through points (20, 0) and (25, 5).

Answers

Answer:

A graph titled Kristine's Savings has money earned on the x-axis and money saved on the y-axis. A line goes through points (20, 0) and (25, 5).

Step-by-step explanation:

If Kristine spends $20 and saves the rest when she gets paid, the amount she saves should always be 20 less than the amount she earned.

The points (20, 0) and (25, 5) accurately represent this, where x is how much she earns and y is how much she saves.

(20, 0) represents her earning $20 and saving $0, because she spends $20 every time she gets paid.

(25, 5) represents her earning $25 and saving $5, because she spent $20.

So, the correct graph is A graph titled Kristine's Savings has money earned on the x-axis and money saved on the y-axis. A line goes through points (20, 0) and (25, 5).

Answer:

d

Step-by-step explanation:

Instructions: Find the missing side. Round your answer to the nearest tenth.
22
58°

Answers

Answer:

x = 19.2

Step-by-step explanation:

tan(58)=x/12

x=12×tan(58)

x=19.2

Answered by GAUTHMATH

if 2/-5 x=-10/x what is the value of x​

Answers

Answer:

± 5

Step-by-step explanation:

2x/-5 = -10/x

2x^2 = 50

x^2 = 25

x = ± 5

Answer:

x=5

Step-by-step explanation:

Start with writing it like 2x/-5= -10/x

Then cross multiply: 2[tex]x^{2}[/tex]= 50

Divide by 2: [tex]x^{2}[/tex]=25

Square root of 25: 5

x=5

NO LINKS OR ANSWERING QUESTIONS YOU DON'T KNOW!!!

Chapter 11 part 2:

What are three different properties of logarithmic functions when encountering the operations of addition, subtraction, and multiplication? Provide an example of each.

Answers

The three main log rules you'll encounter are

log(A*B) = log(A) + log(B)log(A/B) = log(A) - log(B)log(A^B) = B*log(A)

The first rule allows us to go from a log of some product, to a sum of two logs. In short, we go from product to sum. The second rule allows us to go from a quotient to a difference. Lastly, the third rule allows to go from an exponential to a product.

Here are examples of each rule being used (in the exact order they were given earlier).

log(2*3) = log(2) + log(3)log(5/8) = log(5) - log(8)log(7^4) = 4*log(7)

----------------

Here's a slightly more complicated example where the log rules are used.

log(x^2y/z)

log(x^2y) - log(z)

log(x^2) + log(y) - log(z)

2*log(x) + log(y) - log(z)

Hopefully you can see which rules are being used for any given step. If not, then let me know and I'll go into more detail.

Other Questions
solve for F ------- 2/3 + f = 1/5A= 13/15B= 7/15C= - !3/15D= - 7/15 What is the price after taking 25% off $62? I need help on the crossing movie 1. How many Hessians were taken prisoner and how many Americans were killed or wounded at the Battle of Trenton?2. How many months after the American Declaration of Independence was the Battle of Trenton fought? 3. In the Battle of Trenton, the Continentals outnumbered the Hessians almost 2 to 1. The Hessians were surprised and were poorly led. Why was this victory considered so important?4. Where did the Hessians come from and why were they fighting for the British? 5. What would probably have happened to the American Revolution had Washington not crossed the Delaware and engaged the Hessians at Trenton at the end of December, 1776? 6. One of the memorable incidents shown in the film is Washington making a joke at the expense of General Knox, his commander of artillery. The novelist and screenwriter, Howard Fast, found a reference to this in the memoirs of some of the soldiers who were present. What role did Washington's joke play in his leadership at the Battle of Trenton? 7. Describe three actions other than cracking the joke about General Knox that Washington took which showed leadership at Trenton. 8. Washington was described as one of those rare few who, under fire, appeared to be without fear. Why is this important in a military leader? 9. After the battle, the character of General Greene asks the character of General Washington to speak with Colonel Rall before Rall dies. Washington replies: "Do you want me to weep for those b******, men who kill for profit?" General Greene responds that: "Our own cause at its heart is a fight against taxation, is it not? In the end, we all kill for profit, the British and the Hessians and us." Do you agree with the character of General Greene that the American Revolution was fought for money?10. (Be honest; Don't deceive, cheat or steal; Be reliable -- do what you say you'll do; Have the courage to do the right thing; Build a good reputation; Be loyal -- stand by your family, friends and country) The Battle of Trenton shows the Continental troops demonstrating this Pillar. What did they do? Do the two trapezoids in the figure appear to be similar? Why or why not?options:A) They're not similar because only one pair of corresponding angles in the two trapezoids is congruent.B) They're similar because two pairs of corresponding angles in the two trapezoids are congruent.C) They're similar because one pair of corresponding angles in the two trapezoids is congruent.D) They're not similar because two pairs of corresponding angles in the two trapezoids are congruent. But when a long train of abuses and usurpations, pursuing invariably the same Object evinces a design to reduce them under absolute Despotism, it is their right, it is their duty, to throw off such Government, and to provide new Guards for their future security.Such has been the patient sufferance of these Colonies; and such is now the necessity which constrains them to alter their former Systems of Government.The Declaration of Independence,Thomas JeffersonBased on the passage, what is the purpose of the Declaration of Independence?to provide a set of rules and regulations for the United Statesto justify the American colonists desire for freedom from British ruleto establish new settlements and colonies in Americato encourage England to become a democracyIT'S B I JUST DID IT!!!! which statement BEST completes the diagram: colonists bring diseases like smallpox in North America, Colonists believe land can be owned by individual people, American Indian peoples suffer greatly.A- colonists refuse to teach American Indians how to farmB-colonists become the leaders of american indian religious groupsC- colonists introduce steel weapons and guns to american indiansD- colonists force american indians into slavery on plantations HELP ME PLSSS SUMMER SCHOOL A HARD Can you help with this What is the greatest common factor of 10 and 13?ANSWER IN LESS THEN 3 MINS AND I'LL GIVE BRAINLIEST A health club sold ten memberships in one week for total of $3000. If male memberships cost $300 and female memberships cost $300, then how many male memberships and how female memberships were sold? What is the HCF for 20 40 35 Plzz answer this questionParagraph writing Select the correct answer.A student read this excerpt from an argument against school uniforms and agrees with the author. Which statement belowwould help the student defend this argument?excerpt from The Debate over School UniformsSupporters of school uniforms assert that when students wearuniforms, their attendance Improves. They claim that studentswho wear uniforms have more school spirit, which encouragesstudents to attend school more often. In Irving, Texas, schoolshave been able to improve average daily attendance through astrict uniform policy. Though students do not wear officialuniforms, they wear white shirts, khaki pants, and leather shoesthat must meet certain uniform requirements. However, innearby Euless, Texas, no uniforms are required, and yetstudents have higher average daily attendance rates than at thecharter school in Irving. This fact seems to demonstrate thathigher attendance rates have little to do with uniforms. Aschool in Polk County, Florida, has the most restrictive uniformpolicy in the nation. It was enacted in 1999, and since then, theschool has found that its average daily attendance has fallendramatically. In 2000, the school broke a national scholasticabsentee record for the same school year. These results showthat uniforms have no effect on attendance. In fact, theyprovide evidence to the contrary: strict uniform policies have anegative effect on attendance.OA Enforcing school uniforms will improve campus culture.OB.Charter schools and public schools are unfair comparisons in the matter of school uniforms.OC. There is a relationship between uniforms and school attendance.ODThere is no connection between school uniforms and Improved attendanceResetNext what were the disadvantage of the permanent settlement of Bengal Which would increase the reaction rate?O A. Decreasing the reactant concentrationsB. Removing heat from the systemO C. Adding a catalystO D. Lowering the value of k A wire long and with mass is positioned horizontally near the earth's surface and perpendicular to a horizontal magnetic field of magnitude . What current I must flow through the wire in order that the wire accelerate neither upwards nor downwards Please help me. I will mark you as brainliest as soon as possible. The behavior of allosteric enzymes a. does not play any role in feedback inhibition in metabolic pathways b. is strongly dependent on the presence of metal ions c. is related to their ability to hydrolyze themselves d. depends on changes in their quaternary structure on binding of substrates or inhibitors You purchased a stock at a price of $47.52. The stock paid a dividend of $1.55 per share and the stock price at the end of the year was $52.34. What was the total return for the year . Two mutually exclusive projects have projected cash flows as follows:YEAR PROJECT A PROJECT B0 Ksh. -2m Ksh. -2m1 1m 02 1m 03 1m 04 1m 6mRequired:a) Determine the internal rate of return for each project. [2 Marks]b) Determine the net present value for each project at discount rates of 0, 5,10,20,30, and 35 percent. [2 Marks]c) Plot a graph of the net present value of each project at the different discount rates. [2 Marks]d) Which project would you choose? Why? [ 2 Marks]e) What is each projects MIRR if the cost of capital is 12 percent?